logo

部分空間の基底から拡張された基底への線形変換の行列表現 📂線形代数

部分空間の基底から拡張された基底への線形変換の行列表現

定理

$W \le V$を$n$次元 ベクトル空間の$V$の部分空間とする。$\gamma = \left\{ \mathbf{v}_{1}, \dots ,\mathbf{v}_{k} \right\}$を$W$の順序基底とする。$\beta = \left\{ \mathbf{v}_{1}, \dots, \mathbf{v}_{k}, \mathbf{v}_{k+1}, \dots \mathbf{v}_{n} \right\}$を$\gamma$から拡張された$V$の基底という。$T : V \to V$を線形変換とする。それでは、$\beta$に対する$T$の行列表現は次のようなブロック行列である。

$$ \begin{bmatrix} T \end{bmatrix}_{\beta} = \begin{bmatrix} A_{1} & A_{2} \\ O & A_{3} \end{bmatrix} $$

この時、$A_{1} = \begin{bmatrix} T|_{W} \end{bmatrix}_{\gamma}$であり、$T|_{W} : W \to W$は縮小写像、$A_{2}$は$k \times n-k$行列、$A_{3}$は$n-k \times n-k$行列、$O$は$n-k \times k$ゼロ行列である。

証明

$T|_{W} : W \to W$を縮小写像とする。$\begin{bmatrix} T \end{bmatrix}_{\beta} = [t_{ij}]$とする。

$$ \begin{align*} \begin{bmatrix} T \end{bmatrix}_{\beta} = \begin{bmatrix} t_{11} & t_{12} & \cdots & t_{1n} \\ t_{21} & t_{22} & \cdots & t_{2n} \\ \vdots & \vdots & \ddots & \vdots \\ t_{n1} & t_{n2} & \cdots & t_{nn} \end{bmatrix} \end{align*} $$

行列表現を見つけるためには、基底の元がどのようにマッピングされるかを知ればよい。つまり、$\begin{bmatrix} T \end{bmatrix}_{\beta}$の第一列の成分$t_{i1}$は、$T\mathbf{v}_{1}$を$\beta$の線型結合で表した時の$\mathbf{v}_{i}$の係数と同じである。しかし$T \mathbf{v}_{1} = T|_{W}\mathbf{v}_{1}$であるため、

$$ \sum_{i=1}^{n} a_{i1} \mathbf{v}_{i} = T\mathbf{v}_{1} = T|_{W}\mathbf{v}_{1} = \sum_{i=1}^{k} a_{i1} \mathbf{v}_{i} $$

従って、$a_{k+1} = a_{k+2} = \cdots = a_{n} = 0$である。したがって、$1\le i \le k$の時$t_{i1} = a_{i1}$であり、$k \gt i$の時$t_{i1} = 0$である。

$$ \begin{align*} \begin{bmatrix} T \end{bmatrix}_{\beta} = \begin{bmatrix} a_{11} & t_{12} & \cdots & t_{1n} \\ a_{21} & t_{22} & \cdots & t_{2n} \\ \vdots & \vdots & \ddots & \vdots \\ a_{k1} & t_{k2} & \cdots & t_{kn} \\ 0 & t_{k+1,2} & \cdots & t_{k+1,n} \\ \vdots & \vdots & \ddots & \vdots \\ 0 & t_{m2} & \cdots & t_{mn} \end{bmatrix} \end{align*} $$

同じ方法で$k$列目までの成分を全て求めると、以下のようになる。

$$ \begin{bmatrix} T \end{bmatrix}_{\beta} = \begin{bmatrix} a_{11} & a_{12} & \cdots & a_{1k} & t_{1,k+1} & \cdots & t_{1n} \\ a_{21} & a_{22} & \cdots & a_{2k} & t_{2,k+1} & \cdots & t_{2n} \\ \vdots & \vdots & \ddots & \vdots & \vdots & \ddots & \vdots \\ a_{k1} & a_{k2} & \cdots & a_{kk} & t_{k,k+1} & \cdots & t_{kn} \\ 0 & 0 & \cdots & 0 & t_{k+1,k+1} & \cdots & t_{k+1,n} \\ \vdots & \vdots & \ddots & \vdots & \vdots & \ddots & \vdots \\ 0 & 0 & \cdots & 0 & t_{m,k+1} & \cdots & t_{mn} \end{bmatrix} $$

この時、$A_{1} = [a_{ij}] = \begin{bmatrix} T|_{W} \end{bmatrix}_{\gamma}$であり、$A_{2} = \begin{bmatrix} t_{1,k+1} & \cdots & t_{1n} \\ \vdots & \ddots & \vdots \\ t_{k,k+1} & \cdots & t_{kn} \\ \end{bmatrix}$、$A_{3} = \begin{bmatrix} t_{k+11,k+1} & \cdots & t_{k+1,n} \\ \vdots & \ddots & \vdots \\ t_{m,k+1} & \cdots & t_{mn} \\ \end{bmatrix}$とすると、

$$ \begin{bmatrix} T \end{bmatrix}_{\beta} = \begin{bmatrix} A_{1} & A_{2} \\ O & A_{3} \end{bmatrix} = \begin{bmatrix} \begin{bmatrix} T|_{W} \end{bmatrix}_{\gamma} & A_{2} \\ O & A_{3} \end{bmatrix} $$